Announcement

Collapse
No announcement yet.

Not a log, just some general fitness questions

Collapse
X
 
  • Filter
  • Time
  • Show
Clear All
new posts

    Not a log, just some general fitness questions

    Hello, since Bryan/Moneymaker will no doubt grow sick of me annoying him on Skype with constant questioning I thought I'd open this thread if that's cool.

    Basically just have a few questions about my average daily routine to see if there's anything obviously wrong with what I'm doing, I've been trying to get into better shape and generally improve my fitness/lose weight over the last year or so and it's been slow but steady progress with me gradually improving my habits as it's gone on. Atm I weigh ~80.9kg and I'm around 5 foot 11, I've been off drink since February bar one night for my birthday for largely unrelated reasons but it's obv been a positive influence too. I've no idea of my 'starting' weight, but it was a fair bit more than that.

    My exercise routine I've recently slightly updated, 5 days a week I go out jogging. The jogging track is ~8.5km according to Google maps, but I'm not nearly fit enough to jog solidly for the whole thing, so I'm probably jogging for around 6/6.5km of that and walking for the rest. One day a week I'll also go swimming for around 45 minutes, getting around 35 lenghts of a 25 metre pool in doing the breast stroke since I sadly can't do the crawl to save my life. Daily I try get 300 sit ups in and have started doing push ups too, and for a warm up before this I do a routine sent to me by Moneymaker of...Band pull apart (Using a towel unfortunately) - Bw squat - Band dislocates - wall slides - kneeling plank g-ups - bw lunges - mountain climbs, doing around a minute of each.

    Food wise for breakfast I have 2 poached eggs and green tea, for dinner, usually 4/5 hours after breakfast, it varies but say something like fish and instant noodles or chicken and pasta etc for the most part, pretty standard stuff, obviously if I'm out this can vary considerably however. Beyond that I stick mainly to fruit, apples, bananas, strawberries, oranges, grapes, pears etc for snacks whenever I'm hungry between bf and dinner and after dinner, again if I'm out this can vary however.

    So basically, as a daily routine for most days, how is that, with the basic goal to get my weight down to around 70kg? Anything should I be doing differently? Unfortunately I can't commit to a gym membership as I don't plan on being in this country in a few months so that makes that considerably harder. I put todays diet and exercise onto that myfitnesspal site angryball linked too and the results were...


    --------------Calories--Carbs---Fat----Protein
    Totals--------1,247----95------34------21
    Daily Goal-----2,165----298-----71 -----81
    Remaining-----918------203-----37-----60

    It's adjusted my daily goal based on the exercise I've done and what weight I said I want to get too, should I be worried about the fact I've so many calories etc 'remaining' to get to this goal (Today was, if anything, slightly above average for those numbers, especially fat, since I've some leftover remains from a bbq I've been trying to finish off)? Is that a bad thing and should I be eating more as it'd be more +Ev for losing the weight or are those numbers okay? I've no particular timeframe for getting down to 70kg, is this something I should set myself and if so what'd be realistic?

    My main problem is I'm a complete n00b when it comes to the science and technical side of all of this kind of stuff so any help, advice or observations would be much appreciated. Cheers.
    Last edited by Sledgejammer; 28-06-11, 21:24.
    "In the world, there are many kings but there is only one God. I am God, I am El Tren" :{)

    #2
    Get out the scales for your next week of eating, and weigh absolutely everything. You'll be amazed at how poor you are at figuring out how much of things you are eating.

    Exercise, lets up the resistance work, all the situps in the world won't make much of a difference to your overall health and wellbeing imo. Start trying to do some more bodyweight exercises if you're not interested in the gym, there's tonnes of them, and variations of the lot.

    Essentially, there's a few things to think about with exercise that people miss constantly. How often do you see women out "for a walk" teetering along at a mild pace and spending a long time at it? How many of these are slight? Make shit tougher, and it will make you tougher. If you can do something easily, make it harder, once that becomes easy, make it harder again. This is the essence of exercise. Your body will get used to a new "level" very quickly, and if you don't push on, you'll stall and wont get much benefit from it.

    What's your thoughts on lifting weights? Less than 12 months ago I was an utter and complete noob at it, I could barely manage the bar for 5 reps on some exercises, yet now I'm a thirsty bastard for lifting things I shouldn't.

    What are your overall goals? Moneymaker's done brilliantly because he's set realistic (but tough) goals, and stuck to the path rigidly. Performing exercises and cleaning up your diet "just because" is fine, but you'll see far better returns when you challenge yourself.

    Comment


      #3
      Jesus man nowhere near enough protein there imo waiting 4 to 5 hours for next meal after 2 eggs and basically warm water green tea very little nutrients is not good.

      Protein intake is easy to up though a boiled 200gram chicken breast would have 55 gs of protein handy in a salad sandwich curry loads of things.

      I think if you dont up your protein intake youll lsoe weight not just fat but muscle as well esp if your exercising.
      I gram of protein a day per pound of body weight is a good goal to have especially if your doing strength and resistance work
      im sure the more knowledgeable posters might explain better but i think what ive written is correct

      Comment


        #4
        Exercise routine is fairly crap and will make you skinny fat given the amount of food you are eating (which is also way to little).

        Im not sure I have time to write up a proper reply now but I will the minute I get some free time.
        This may or may not be an original thought of my own.
        All efforts were made to make this thought original but with the abundance of thoughts in the world the originality of this thought cannot be guaranteed.
        The author is not liable for any issue arising from the platitudinous nature of this post.

        Comment


          #5
          Basically just have a few questions about my average daily routine to see if there's anything obviously wrong with what I'm doing, I've been trying to get into better shape and generally improve my fitness/lose weight over the last year or so and it's been slow but steady progress with me gradually improving my habits as it's gone on. Atm I weigh ~80.9kg and I'm around 5 foot 11, I've been off drink since February bar one night for my birthday for largely unrelated reasons but it's obv been a positive influence too. I've no idea of my 'starting' weight, but it was a fair bit more than that.
          5'11''? really? I never remember you being much taller than me? You're def not fat fat if you are 5'11 and 81kg. Im 5'9'' I think and 81kg and I aint fat so you cant be. That said, you are prob skinny fat, like most people.


          I'm going to be harsh because I know you!!

          My exercise routine I've recently slightly updated, 5 days a week I go out jogging. The jogging track is ~8.5km according to Google maps, but I'm not nearly fit enough to jog solidly for the whole thing, so I'm probably jogging for around 6/6.5km of that and walking for the rest. One day a week I'll also go swimming for around 45 minutes, getting around 35 lenghts of a 25 metre pool in doing the breast stroke since I sadly can't do the crawl to save my life. Daily I try get 300 sit ups in and have started doing push ups too, and for a warm up before this I do a routine sent to me by Moneymaker of...Band pull apart (Using a towel unfortunately) - Bw squat - Band dislocates - wall slides - kneeling plank g-ups - bw lunges - mountain climbs, doing around a minute of each.
          Jogging is good. Cardio fitness and the dedication to actually get out there are very important. There are better ways to get it done though. 300 situps? Christ. Utter waste of time. You do Moneymakers routine before the situps and push ups?

          At your weight and height MM's routine is fairly useless from a 'better body' point of view. Its similar to doing a cardio workout rather than a resistance workout.

          Keep the swimming, good for recovery.

          I'm also assuming your goal is fatloss?

          Food wise for breakfast I have 2 poached eggs and green tea, for dinner, usually 4/5 hours after breakfast, it varies but say something like fish and instant noodles or chicken and pasta etc for the most part, pretty standard stuff, obviously if I'm out this can vary considerably however. Beyond that I stick mainly to fruit, apples, bananas, strawberries, oranges, grapes, pears etc for snacks whenever I'm hungry between bf and dinner and after dinner, again if I'm out this can vary however.
          Eggs for breakfast. Thats a win.

          Do you eat lunch? or brunch? or linner? I eat all 5. You should too.

          Lunch should be meat and green veg.

          Dinners crap. Meat and green veg please. Obv add in onions, peppers, green beans etc by all means( this goes for lunch too). Also anythnig spicey is a win (makes the blandness taste nicer)

          Fruit, by and large, is pretty crap for fatloss. I try to explain this to people but such is the 'but fruits are so healthy' mindset among the greater public its tough to get it through.

          So basically, as a daily routine for most days, how is that, with the basic goal to get my weight down to around 70kg? Anything should I be doing differently? Unfortunately I can't commit to a gym membership as I don't plan on being in this country in a few months so that makes that considerably harder. I put todays diet and exercise onto that myfitnesspal site angryball linked too and the results were...
          Why 70kg?

          If you cant commit to a gym membership buy some weights or find a pay as you go gym? Otherwise you're kidding yourself. You can do all the cardio and BW exercises in the world and even if you get lean, you'll be very small.


          --------------Calories--Carbs---Fat----Protein
          Totals--------1,247----95------34------21
          Daily Goal-----2,165----298-----71 -----81
          Remaining-----918------203-----37-----60

          It's adjusted my daily goal based on the exercise I've done and what weight I said I want to get too, should I be worried about the fact I've so many calories etc 'remaining' to get to this goal (Today was, if anything, slightly above average for those numbers, especially fat, since I've some leftover remains from a bbq I've been trying to finish off)? Is that a bad thing and should I be eating more as it'd be more +Ev for losing the weight or are those numbers okay? I've no particular timeframe for getting down to 70kg, is this something I should set myself and if so what'd be realistic?
          Essentially, you need to be eating under 500cals of your 'maintenence' level. At the end of the day calories are all that matter for WEIGHT loss. If you eat too little though, you're body will tell you to GTFO and stubbornly hold onto fat.

          After calories, WHAT you eat is what matters for FAT loss.

          Keep it fresh, green and meaty. High Protein, High Fat, Low Carbs. Aim for 2200 cals a day. (estimating it off my own weight, same as yours, I aim for 2500 but Ive prob a bit more lean mass than you)

          If I read this right that sites telling you you're daily target of carbs is 300g? and only 81g protein? Crazy site. If you reversed it you'd be some way to better off.

          Fat is good. Dont villify it.

          Yes set a timeframe and yes its important. 2 stone to lose effectively, Id say 12 weeks is a fair timeframe.


          My main problem is I'm a complete n00b when it comes to the science and technical side of all of this kind of stuff so any help, advice or observations would be much appreciated. Cheers.
          Like I say to everyone, loads of very knowledgeable posters on these topics so ask away in here.

          I'm actually in the process of doing a sticky for this place covering everything but its taking me abit longer than I thought as Im pretty busy.
          This may or may not be an original thought of my own.
          All efforts were made to make this thought original but with the abundance of thoughts in the world the originality of this thought cannot be guaranteed.
          The author is not liable for any issue arising from the platitudinous nature of this post.

          Comment


            #6
            That 'routine' of mine is my warm-up. Takes 5 minutes to do 2 circuits then the proper work begins for 45 minutes or so.

            I don't get sick of discussing this stuff, as i've said it's become a bit of an obsession really. But it's good to start a thread here as there's plenty of guys far more knowledgeable then me.

            Listen to Theresa. Words of wisdom.

            If you can't join a gym buy some weights and a bench if possible.

            Stick to berries, other fruits aren't great really. Get tons of veg and lean and high protein meats into ya.

            1500-1800 calories should be spot on. Under-eating is a recipe for disaster, eat healthy and frequently.

            Nuts are fantastic to snack on in moderation. A tablespoon of peanut butter will do you no harm either.

            Set yourself monthly targets. Don't just live this lifestyle for the sake of it. Challenge yourself and test your capabilities. You'll be surprised at how rewarding it is.

            Keep up the good work, i'd guess you were the guts of 120kg(remember the mix game at the Voodoo, we were both such droolers, ugh) so you've done a great job.

            Comment


              #7
              I haven't got much to add, Theresa pretty much said exactly what I was thinking. Agree 100% with his post.


              --------------Calories--Carbs---Fat----Protein
              Daily Goal-----2,165----298-----71 -----81


              That target/goal is terrible, where did it come from. You are pretty much the same weight as me, slightly taller. And I'd consider that to be a pretty bad day for me.

              I'd rather go with something like;
              --------------Calories--Carbs---Fat----Protein
              Daily Goal-----2,000----140-----80 -----180

              Comment


                #8
                Originally posted by Moneymaker View Post
                1500-1800 calories should be spot on. Under-eating is a recipe for disaster, eat healthy and frequently.

                Nuts are fantastic to snack on in moderation. A tablespoon of peanut butter will do you no harm either.

                .
                Just to clairfy the discrepency between mine and MM's calorie recommendation:

                I said 2200 for the diet (2200). Exercise should bring it down 3/400. 1800 altogether.

                MM obviously gave you the figure you should be aiming for in total (exercise+diet).

                Anyway, this perfectly illustrates the point that In order to change weight +- then diet is the main thing. Keeping exercise consistent makes it alot easier.
                This may or may not be an original thought of my own.
                All efforts were made to make this thought original but with the abundance of thoughts in the world the originality of this thought cannot be guaranteed.
                The author is not liable for any issue arising from the platitudinous nature of this post.

                Comment


                  #9
                  Yup I reckon its around 80/20 Diet/exercise in whether or not you'll lose weight.

                  And yeah, 1500-1800 after exercise is what I would go for. Eat a little bit more on rest days. Your body will need the nutrients for repair and muscle growth.

                  Comment


                    #10
                    Cheers for the advice lads, the sheer volume of free advice you can get on just about any subject on here is insane. Haven't got the time to go through them all individually so will do a longer reply later, but based on what you're saying I've been looking into a few gyms that I can get a few months membership out of and it seems easier than I expected, so that'll hopefully be a runner in the next few days then when I get to Toronto get a full time membership in one. I'll go into more specifics later, mainly about the diet, since I've still got alot of questions but thanks everyone for taking the time out to reply.
                    "In the world, there are many kings but there is only one God. I am God, I am El Tren" :{)

                    Comment


                      #11
                      will follow this thread. i'm interested in the diet tips especially.

                      Comment


                        #12
                        Right, probably alot of tl;dr here, I'm ignoring MM since I was talking to him on Skype

                        Originally posted by Emmet View Post
                        Get out the scales for your next week of eating, and weigh absolutely everything. You'll be amazed at how poor you are at figuring out how much of things you are eating.

                        Exercise, lets up the resistance work, all the situps in the world won't make much of a difference to your overall health and wellbeing imo. Start trying to do some more bodyweight exercises if you're not interested in the gym, there's tonnes of them, and variations of the lot.
                        Have looked at a few of these and will start doing a routine of them from now on, cheers for the advice, though as I've said hopefully I'll actually be able to get the gym sorted after all which would be a big help also.

                        Essentially, there's a few things to think about with exercise that people miss constantly. How often do you see women out "for a walk" teetering along at a mild pace and spending a long time at it? How many of these are slight? Make shit tougher, and it will make you tougher. If you can do something easily, make it harder, once that becomes easy, make it harder again. This is the essence of exercise. Your body will get used to a new "level" very quickly, and if you don't push on, you'll stall and wont get much benefit from it.

                        What's your thoughts on lifting weights? Less than 12 months ago I was an utter and complete noob at it, I could barely manage the bar for 5 reps on some exercises, yet now I'm a thirsty bastard for lifting things I shouldn't.

                        What are your overall goals? Moneymaker's done brilliantly because he's set realistic (but tough) goals, and stuck to the path rigidly. Performing exercises and cleaning up your diet "just because" is fine, but you'll see far better returns when you challenge yourself.
                        I've never really tried lifting weights but it's certainly something I'll start trying now, hopefully can get everything sorted in a couple of days and get in touch with a personal trainer there who can tell me exactly what I should be doing vis a vie weights. I agree completely with your sentiment on pushing yourself though, and is something I've been doing. My goal is just general increased fitness, losing weight and remaining healthy, my problem with setting specific goals is I'm not sure exactly what I should be aiming for in terms of specific targets.

                        Originally posted by Theresa View Post
                        5'11''? really? I never remember you being much taller than me? You're def not fat fat if you are 5'11 and 81kg. Im 5'9'' I think and 81kg and I aint fat so you cant be. That said, you are prob skinny fat, like most people.

                        I'm going to be harsh because I know you!!
                        Tbh I've not measured in ages, I thought I was 5'11 but that may be a little off. Also harshness is exactly what's needed, the harsher the advice chances are the more routine changing it'll be therefore the better in the long run, so it's all good

                        Jogging is good. Cardio fitness and the dedication to actually get out there are very important. There are better ways to get it done though. 300 situps? Christ. Utter waste of time. You do Moneymakers routine before the situps and push ups?

                        At your weight and height MM's routine is fairly useless from a 'better body' point of view. Its similar to doing a cardio workout rather than a resistance workout.

                        Keep the swimming, good for recovery.

                        I'm also assuming your goal is fatloss?
                        I do it as a warm up yeah, just for 5 minutes as he said. Yeah my overall goal is fatloss/reducing weight alright, so I'm going to start switching to more resistance work and hitting the gym like I said, I don't know exactly what routine I'll have once I do, probably talk to the personal trainers there and get a better idea, obviously wanna balance it out alot better and not have it so cardio intense.

                        Eggs for breakfast. Thats a win.

                        Do you eat lunch? or brunch? or linner? I eat all 5. You should too.

                        Lunch should be meat and green veg.

                        Dinners crap. Meat and green veg please. Obv add in onions, peppers, green beans etc by all means( this goes for lunch too). Also anythnig spicey is a win (makes the blandness taste nicer)

                        Fruit, by and large, is pretty crap for fatloss. I try to explain this to people but such is the 'but fruits are so healthy' mindset among the greater public its tough to get it through.
                        I'd just been having fruit for lunch and yeah the logic went no deeper than 'fruit=good' since I didn't really have any better understanding to go with. Today however I'd the standard breakfast, for lunch I'd a noodle stir fry with vegetables and meat, chicken and peas for dinner and strawberries for snacks, is that already looking like a better day? I'm going to be going shopping tomorrow so will stock up on more stuff. MM recommend getting some protein shakes and having a basic idea of breakfast, workout, shake, lunch, snack, dinner and yeah basically echoing what you were saying about what kind of dinners and lunches I should be having. I'll have to get some of that protein shake that was being discussed in his thread.

                        Why 70kg?

                        If you cant commit to a gym membership buy some weights or find a pay as you go gym? Otherwise you're kidding yourself. You can do all the cardio and BW exercises in the world and even if you get lean, you'll be very small.
                        70k was simply a target I'd set for no real reason other than I'd read somewhere it was within the 'ideal' weight for someone my height and I wanted a target to aim for, like I said, I lack a basic understanding about most of these things. The gym thing will hopefully be sorted, I think I've found a pretty solid deal that really suits me like I said, so that should help the exercise routine alot.

                        Essentially, you need to be eating under 500cals of your 'maintenence' level. At the end of the day calories are all that matter for WEIGHT loss. If you eat too little though, you're body will tell you to GTFO and stubbornly hold onto fat.

                        After calories, WHAT you eat is what matters for FAT loss.

                        Keep it fresh, green and meaty. High Protein, High Fat, Low Carbs. Aim for 2200 cals a day. (estimating it off my own weight, same as yours, I aim for 2500 but Ive prob a bit more lean mass than you)
                        Right, all this taken on board, but what is the 'maintenance' level? Beyond that this is all good advice.

                        Yes set a timeframe and yes its important. 2 stone to lose effectively, Id say 12 weeks is a fair timeframe.
                        This seems like as good a target as any, so I'll shoot for that.

                        Like I say to everyone, loads of very knowledgeable posters on these topics so ask away in here.

                        I'm actually in the process of doing a sticky for this place covering everything but its taking me abit longer than I thought as Im pretty busy.
                        That'd make a fantastic read, and I really do appreciate the time taken out by yourself and everyone else to answer these things, sorry for all the stupid questions but at least it'll probably lead to some easy copy and pasting for the sticky.
                        "In the world, there are many kings but there is only one God. I am God, I am El Tren" :{)

                        Comment


                          #13
                          I'm pretty (read: very) skeptical of personal trainers in large box gyms. They frequently havent got a clue and are being taught old methods and theories on their training courses. They are also usually so far off with nutrition that its not safe.

                          That said, be interesting to see where you go and what advice you get. Ask away as Id like to see just how bad it is. I'd love to be proven wrong though.


                          Resistance:
                          Compound exercises (those that use more than one muscle/muscle group) are imo, the best weight exercises to do for a variety of reasons.
                          • More bang for your buck (in terms of metabolic upheaval),
                          • Shorter time in the gym (as opposed to endless 5sets 8reps of exercises like tricep kickbacks),
                          • You'll be more tired (which is a plus as you feel like your getting shit done


                          Starting Strength is a good beginner programme, it'll stand you well for the first while. Google 'Starting Strength Mark Rippetoe'- its primarily a strength training programme but if you do it two days a week and then the third day do some metabolic conditioning sessions (essentially where you do circuits of fast, high intensity exercises) you'll see very good results.

                          I'd also recommend this because; since you are new to lifting your gonna have a serious hormonal, muscle building spike. We call it 'newbie gains'. This programme will likely get the best from that. As an aside, and to anyone more knowledgeable reading; Im sure their are other ways to do this and other great programmes but this one is simple, easy to follow and gets good results imo. Of course any sort of routine will get results but having a structured, proven beginner programme will get you BETTER results.

                          Get Marks book (someone here might have it, I used to on my old laptop) and read it. Its a bit heavy but it'll ingrain some of the core movements in to you. Thatll stand to you. (reading the book isnt a must or isnt neccessary but I think its beneficial). If you've no interest, the programme is available on the net or from someone here.

                          Regarding Metabolic Conditioning:

                          Things like:
                          10 Push ups
                          10 Crunches
                          100 Sprint
                          20 Bodyweight squats

                          Repeat 5 times. 2 mins rest between each set.

                          Or
                          Skipping 30 secs
                          Plank 30secs
                          Shuttle run
                          10 burpees

                          as above. Things like that.

                          Diet:

                          This is the easy part and people LOVE to complicate it.

                          Follow these basic rules and adjust calorie intake for specific goals.


                          Read the nutritional info on packets.
                          Keep food as fresh and meaty as possible.
                          Fuck sugar. It causes HAVOC with you.
                          Meat and green vegetables. (memorise that)

                          Breakfast: Eggs. easy right? You can have some meat too, a sausage and two eggs. is approx 500 cals. 2 rasher and 2 eggs is the same. Use your noggin. Dont FRY them. Grill them.

                          Lunch: A grilled burger/chicken breast/Salmon Fillet/Steak whatever with some spinach leaves, risotto and peppers. Meat and green veg. My lunch right now is a Kangaroo Steak and copious amounts of spinach leaves.

                          Dinner: Same as lunch. Keep it simple.

                          Those should be the mainstay meals, 500-600 cals each. Add another 2 snack meals (nuts, berries, jerky, leftover meats) of 250-300 cals and your are flying. Learn what cals are in what.

                          I usually cook up a massive pot of meat an veg at the start of teh week and use it for lunches during the week. No excuses. Handy on the pocket too.

                          Supplements:

                          Protein: Not essential, and most people will give the "you dont need it, get protein from wholefoods and your diet"- which is fair, but wholly impractical. Its cheap(relatively) and so very very handy to have.

                          Fish Oil Tabs: A MUST. Need fat to lose fat. Fish oil is THE best fat. A very quick summary of their benefits:

                          Support healthy blood pressure and circulatory systems.
                          Maintain/Regulate Cholestorol levels.
                          Predominant fatty acid in the nervous system (for lifting big weights) and the brain structure.
                          Great for joint and eye health.

                          Maybe get a multi vitamin too just in case. I never used to but saw it advised and guess it makes sense.


                          With ALL this in mind; follow it MOST of the time. 80/20 rule is usually bandied about. Its probably about right. You need a dirty burger every now and then imo. Every now and then mind. Use your discretion.

                          Like I say to everyone, if you've any questions (and again, this is directed towards anyone reading) just ask.
                          Last edited by Theresa; 29-06-11, 22:12.
                          This may or may not be an original thought of my own.
                          All efforts were made to make this thought original but with the abundance of thoughts in the world the originality of this thought cannot be guaranteed.
                          The author is not liable for any issue arising from the platitudinous nature of this post.

                          Comment


                            #14
                            Thats actually the bones of a sticky right there. Will prob just edit that with links/more info later and ask someone to stick it up.
                            This may or may not be an original thought of my own.
                            All efforts were made to make this thought original but with the abundance of thoughts in the world the originality of this thought cannot be guaranteed.
                            The author is not liable for any issue arising from the platitudinous nature of this post.

                            Comment


                              #15
                              I haven't finished reading through the thread yet, so cool beans if someone's already suggested something similar, but there's a shit load of BW metcons you can do at home that are great for weight loss with little or no equipment, e.g.

                              5 rounds of - 15 double unders (30 single unders if you need to)15 box hops (15 squat jumps if a step is not available)15 press ups, 15 squats 15 sit ups - f...


                              (warning, guy is wearing terrible pants)

                              That guy has shitloads of these type of workouts on youtube, posts over on boards as Transform. You can do a ton of stuff like that at home.

                              Buy a pullup bar as well. I put one in the door of my jacks, every time I go for a whizz try and pump out a few pullups, every little helps.

                              We should probably get this thread stickied for people who don't want to keep a log but have questions to ask...
                              Last edited by Keane; 30-06-11, 10:27.

                              Comment


                                #16
                                That's absolutely excellent stuff Steve, have taken it on board and changed my diet accordingly and also that book is winging it's way to me as we speak. Hopefully the gym stuff comes through, I'm waiting to hear back from one atm.

                                Cheers for the link Keane, have been looking into these kind of exercises in the meantime between now and getting into the gym and despite his questionable pants that's exactly what I was looking for, cheers again to all who've replied.
                                "In the world, there are many kings but there is only one God. I am God, I am El Tren" :{)

                                Comment


                                  #17
                                  Originally posted by Sledgejammer View Post
                                  That's absolutely excellent stuff Steve, have taken it on board and changed my diet accordingly and also that book is winging it's way to me as we speak. Hopefully the gym stuff comes through, I'm waiting to hear back from one atm.
                                  Bleurgh I meant to say earlier I have the starting strength PDF and DVD that I could have gotten my cousins to give you.

                                  The DVD is worth getting anyway, remind me later to dig it out...

                                  Comment


                                    #18
                                    Originally posted by Keane View Post
                                    Bleurgh I meant to say earlier I have the starting strength PDF and DVD that I could have gotten my cousins to give you.

                                    The DVD is worth getting anyway, remind me later to dig it out...
                                    No need Keane, I've some cousins of my own who are very kindly donating the entire set to me
                                    "In the world, there are many kings but there is only one God. I am God, I am El Tren" :{)

                                    Comment


                                      #19
                                      Originally posted by Keane View Post
                                      We should probably get this thread stickied for people who don't want to keep a log but have questions to ask...
                                      +1

                                      I'd often like to ask a question on something without having set up a log. Think a general Q&A thread would be a brilliant addition.

                                      Comment


                                        #20
                                        Lets use this as the general Q+A so, until we get one, and then some modley activity will move it in there.

                                        I've been in ketosis for last 5 days, shits are tough going.
                                        How can I up my fibre intake without getting too carby?

                                        Comment


                                          #21
                                          Originally posted by Emmet View Post
                                          Lets use this as the general Q+A so, until we get one, and then some modley activity will move it in there.

                                          I've been in ketosis for last 5 days, shits are tough going.
                                          How can I up my fibre intake without getting too carby?
                                          A fibre supplement surely?
                                          fibrosine is one i heard of

                                          Comment


                                            #22
                                            Originally posted by Emmet View Post
                                            Lets use this as the general Q+A so, until we get one, and then some modley activity will move it in there.

                                            I've been in ketosis for last 5 days, shits are tough going.
                                            How can I up my fibre intake without getting too carby?
                                            Psyllium Husks. You'll get them in most health shops and probably MnS, or maybe Tesco.

                                            Perfect for you. I've been using it/them a fair bit, just pop it in the protein shake/water and drink it.

                                            Be careful not to let it stand too long though. These yokes absorb all the water.
                                            This may or may not be an original thought of my own.
                                            All efforts were made to make this thought original but with the abundance of thoughts in the world the originality of this thought cannot be guaranteed.
                                            The author is not liable for any issue arising from the platitudinous nature of this post.

                                            Comment


                                              #23
                                              Originally posted by Emmet View Post
                                              Lets use this as the general Q+A so, until we get one, and then some modley activity will move it in there.
                                              There should be a sticky up soon, with a lot of general stuff covered, that can do for questions once running.
                                              Moving these might be a problem, as the timestamp will mess the the first posts in the sticky, its best to keep the basic info posts where they can be found

                                              Originally posted by Theresa View Post
                                              Psyllium Husks. You'll get them in most health shops and probably MnS, or maybe Tesco.
                                              LOL once again you pop up with my exact answer.
                                              Ditto, I use psyllium husk too. It's actually a god idea for anyone to have a serve of it daily for a week or two, will clear the system and prob drop a bulge around the waist.

                                              If on keto, take it with shakes or on its own with water. It's not a nice texture, I use to add sugar free flavouring to it to make it easier. But in general you mix it and down it. 10-30g a day imo
                                              Originally posted by SICKPUPPY View Post
                                              A fibre supplement surely?
                                              fibrosine is one i heard of
                                              A lot of fibre supps are simple psyllium husk, process and sold at about 1000% markup

                                              Save money, just buy the raw husk, not sachets, not pills.
                                              It comes in a bag and looks a bit like very rough wholemeal flour

                                              Comment


                                                #24
                                                Agree with Theresa regards personal trainers in commercial gyms. They'll usually give a one-size fits all kinda workout which is basically a few minutes on each machine, machine weights and some core work. They do this because they're not getting paid for the advice, it's much easier/quicker and most don't really care about you tbh. Teaching the compound lifts correctly takes a bit of time.

                                                One thing missing from Theresa's post is flexibility and mobility issues. This is a big problem for most people (and probably another reason why PTs try to avoid teaching the big lifts). The fact of the matter is that most people can't perform the big lifts correctly starting off. To make it worse, it's tough for a beginner to realise that they don't have the flexibility when they're looking in the mirror performing the movements themselves. Getting someone knowledgeable to sort you out or taping your lifts and sticking them on here is a good way to learn. Always remember that technique is key. In terms of solving mobility problems, I'd recommend two things. (1) Magnificent mobility = dvd and (2) Assess and correct = book and dvd package. The book off assess and correct is brilliant. It's like having a physio to refer to 24/7. Foam rolling is massively important too.

                                                I haven't even mentioned the time it takes your brain to learn the motor pathways to perform the compound movements efficiently and consistently but that's for another day. That's another reason why it's important to get the technique right at the beginning I suppose. It's hard to rewire your brain with the correct technique if you've been doing a movement wrong for a few months or beyond.

                                                The big lifts are an art form. There's so many little nuances that can make your lift more efficient. It's interesting as it's akin to constantly trying to solve an ever-changing puzzle where each jigsaw piece is a possible weakness that can lead to a breakdown in technique. Take falling forward in the squat for example. There's so many possible causes. Weak abs/glutes/hammies/upper back, upper back/abs not being kept tight enough, elbows not under the bar enough, not holding your back arch for long enough, incorrect breathing technique, etc. The list goes on and on. I find it very rewarding when you identify a weakness and correct it.

                                                Also, when you get to a certain point, the workouts become mental as well as physical. You can't just turn up and start lifting. You have to get yourself prepared mentally for the challenge at hand. Positive thoughts are crucial and you must always be alert. It's very easy to go into auto mode where you're just performing the movements. I'm guilty of it myself way too often. Always think about what you're doing and why you're doing it. Verbal cues help a ton.

                                                I'll stop my rambling now as I need to sleep.

                                                Comment


                                                  #25
                                                  Originally posted by Lurker23 View Post
                                                  Also, when you get to a certain point, the workouts become mental as well as physical. You can't just turn up and start lifting. You have to get yourself prepared mentally for the challenge at hand. Positive thoughts are crucial and you must always be alert. It's very easy to go into auto mode where you're just performing the movements. I'm guilty of it myself way too often. Always think about what you're doing and why you're doing it. Verbal cues help a ton.
                                                  Agree with everything you said.

                                                  The above is so crucial though, get angry in the gym your there to really really work and push yourself. Mental thoughts are so important.

                                                  Comment


                                                    #26
                                                    What are peoples' thoughts on how much water you ought to be drinking per day?

                                                    Comment


                                                      #27
                                                      I typically drink 3-4 litres per day.

                                                      I'm pretty much constantly sipping on water.

                                                      Keeps you nice and hydrated, flushes a lot of nasty shit out of your system, keeps your muscles working efficiently, boosts your metabolic rate and it's great to drink water before a meal as it will help portion control.

                                                      Cliffs, water = the nuts.

                                                      Comment


                                                        #28
                                                        just keep drinking it. If you're in work, have litre bottle at your desk and keep sipping away.

                                                        You get up out of your chair more often to leak and refill, so it's (a tiny bit) beneficial in that way, and it also makes everything easier in your body tbh.

                                                        If people only changed one thing in their diet and drank water all day and dropped soft drinks, they'd be doing well imo.
                                                        A significant and simple change.

                                                        If your pee's not clear, the weight wont disappear.
                                                        (c)

                                                        Foodies, I have 2kg of mince beef that I'm lost with how to cook.
                                                        I need no carbs / low low carb dinners for it.
                                                        My usual mince abilities are all carbo loaded, lasagne, chilli with rice, shepards pie etc.

                                                        Someone inspire me!

                                                        Comment


                                                          #29
                                                          Originally posted by Emmet View Post
                                                          just keep drinking it. If you're in work, have litre bottle at your desk and keep sipping away.

                                                          You get up out of your chair more often to leak and refill, so it's (a tiny bit) beneficial in that way, and it also makes everything easier in your body tbh.

                                                          If people only changed one thing in their diet and drank water all day and dropped soft drinks, they'd be doing well imo.
                                                          A significant and simple change.

                                                          If your pee's not clear, the weight wont disappear.
                                                          (c)

                                                          Foodies, I have 2kg of mince beef that I'm lost with how to cook.
                                                          I need no carbs / low low carb dinners for it.
                                                          My usual mince abilities are all carbo loaded, lasagne, chilli with rice, shepards pie etc.

                                                          Someone inspire me!
                                                          I usually just make a chilli with a shitload of vegetables and just don't have any rice with it. If you put enough variety of veg I don't see why you'd need any rice really.

                                                          Recipe (fwiw) is here.

                                                          Comment


                                                            #30
                                                            yeah did that with the other KG.

                                                            Can't hack any more chilli though.

                                                            Lidl had 500g of mince for £1 last weekend, all my freezer has is mince...

                                                            Comment


                                                              #31
                                                              Originally posted by Emmet View Post
                                                              yeah did that with the other KG.

                                                              Can't hack any more chilli though.

                                                              Lidl had 500g of mince for £1 last weekend, all my freezer has is mince...
                                                              Fajitas?

                                                              All you really need is cumin, paprika, salt and pepper. Again just don't bother with the wraps obv.

                                                              One of my friends makes a shepherds pie effort using nothing but mince, veg and this stuff:



                                                              Fries the mince and veg, adds about half a jar of the herbs then bangs it in the oven for about ten minutes with some mash on top. Just leave out the mash and you could be onto a winner.

                                                              I assume the herbs themselves are just thyme, oregano and garlic or something, but whatever that particular blend is is very good anyway.

                                                              Comment


                                                                #32
                                                                Originally posted by Emmet View Post
                                                                yeah did that with the other KG.

                                                                Can't hack any more chilli though.

                                                                Lidl had 500g of mince for £1 last weekend, all my freezer has is mince...
                                                                How about some sort of stuffed peppar dish. Chop it in half, take out all the white shit out to leave the hollow, boil for a few mins to soften. Stuff it with mince beef, onions, bean sprots ect... and make some sort of homemade salsa to liven it up. Surprisingly good
                                                                Last edited by The Situation; 01-07-11, 14:16.
                                                                Profit before people.

                                                                Comment


                                                                  #33
                                                                  Im a big fan of bolognase minus the pasta. Reminds me of eating the left overs in college although i leave the brown bread taost and too much butter out . . .
                                                                  Also, love fajitas - goin to have that tonight with mushrooms and salsa, cut the cheese out

                                                                  Was reading up on ways to eat salsa minus the tortilla chips, as its reasonably healthy and delicious

                                                                  The 2 main options i came across are : hard boiled eggs, halfed, with the yoke removed and salsa put it
                                                                  Carrots used as dip for it
                                                                  (also the baked potatoe option, which im ignoring)
                                                                  GAA News Website

                                                                  Comment


                                                                    #34
                                                                    I take it that fresh pasta with Ham and cheese in them is a no no?

                                                                    Comment


                                                                      #35
                                                                      for weight loss? correct, not advised!
                                                                      GAA News Website

                                                                      Comment


                                                                        #36
                                                                        make a shepherds pie emmett and substitute the potatoes for cooked and mashed cauliflower. Also makes a good substitute for Rice when cooked and shredded
                                                                        Low fee Euro/UK money transfer, 1st transfer free through my referral
                                                                        https://transferwise.com/u/bfa0e

                                                                        Comment


                                                                          #37
                                                                          that sounds like a great call! The oul brocolli and cauliflower mash, hape of meat, and a load of cheese.

                                                                          Nice one

                                                                          Comment


                                                                            #38
                                                                            Right, looking at trying to get a bit more creative in the gym as am on a cut, and lifting big is tough going (even though I hit 3 PBs this week) when you just don't have any gas. I'm trying to burn out after my lifts, so am doing 3x5 of A - Squat, OHP, Deadlift, B - Squat, Bench, Inverted Row. Followed by a "whatever's around" metcon.

                                                                            Basically the gym has a few floors, and there's a yoga-esque "ghey" gym that has some "ghey weights", basically rubber weights and 2.5kg bars for them.
                                                                            SPOILER

                                                                            So I've been setting up a barbell complex/metcon after every session. Just trying to figure out if there are any other things I can add in to mix it up, and make sure I'm hitting enough muscles.

                                                                            Basically throw 20/25kg on the bar and do as many of these in a cycle until I am on the verge of puking.
                                                                            Exercises include
                                                                            Clean and Press (10)
                                                                            Barbell Rows (15)
                                                                            Bench Press (15)
                                                                            Squats (15)
                                                                            Jumps (to bench) (20)
                                                                            Press Ups (15)
                                                                            Burpees (10)
                                                                            Thrusters (10) - 10kg is the biggest Medicine Ball / Kettlebell hybrid

                                                                            Usually do the reps in the brackets, and do 3 exercises, 30 second break, rinse and repeat until near death.

                                                                            Anything else I can throw onto this? Or does it look okay?

                                                                            Comment


                                                                              #39
                                                                              So I'm starting in the gym tomorrow...

                                                                              What kind of routine should I be looking to do in the gym, keep in mind n00b status here? Also how many days a week should I be going, and how much things like jogging/swimming etc should I be looking to do beyond that, and do I do those on the same days as the gym if I've time or on the days inbetween?
                                                                              "In the world, there are many kings but there is only one God. I am God, I am El Tren" :{)

                                                                              Comment


                                                                                #40
                                                                                How did you get on in the gym Sledge?

                                                                                Thoughts on chewing gum lads? Trivial or a no-no?

                                                                                Comment


                                                                                  #41
                                                                                  Originally posted by Keane View Post
                                                                                  Thoughts on chewing gum lads? Trivial or a no-no?
                                                                                  Very trivial I would think.

                                                                                  Comment


                                                                                    #42
                                                                                    beyond trivial. Don't swallow it though.

                                                                                    Someone answer my metcon question please. Ended up throwing some EZ Curls and Crucifixes in last night, and my arms are in bits today, probably not the right exercises to be doing as part of that...

                                                                                    Comment


                                                                                      #43
                                                                                      Originally posted by Keane View Post
                                                                                      How did you get on in the gym Sledge?
                                                                                      Solid first day, was mainly focusing on getting my technique right with the help of one of the people there but lifting things is pretty fun...
                                                                                      "In the world, there are many kings but there is only one God. I am God, I am El Tren" :{)

                                                                                      Comment


                                                                                        #44
                                                                                        Theres no chance in hell you'll even get through one circuit of that on you're first few times Emmet.

                                                                                        If you even get to the Burpees, they'll be the ones to make you puke. You're much better of picking 3 and circuiting them until you've some sort of conditioning built up.
                                                                                        This may or may not be an original thought of my own.
                                                                                        All efforts were made to make this thought original but with the abundance of thoughts in the world the originality of this thought cannot be guaranteed.
                                                                                        The author is not liable for any issue arising from the platitudinous nature of this post.

                                                                                        Comment


                                                                                          #45
                                                                                          Originally posted by Keane View Post
                                                                                          Thoughts on chewing gum lads? Trivial or a no-no?
                                                                                          Ah come on Keane.
                                                                                          This may or may not be an original thought of my own.
                                                                                          All efforts were made to make this thought original but with the abundance of thoughts in the world the originality of this thought cannot be guaranteed.
                                                                                          The author is not liable for any issue arising from the platitudinous nature of this post.

                                                                                          Comment


                                                                                            #46
                                                                                            Originally posted by Theresa View Post
                                                                                            Theres no chance in hell you'll even get through one circuit of that on you're first few times Emmet.

                                                                                            If you even get to the Burpees, they'll be the ones to make you puke. You're much better of picking 3 and circuiting them until you've some sort of conditioning built up.
                                                                                            I basically do first set of three, take 30 second breather, second set of 3, 30 second breather, rinse and repeat until tears and puking commences.

                                                                                            Just trying to find some other varieties to stick in.

                                                                                            Yes, Burpees are gruesome.

                                                                                            Comment


                                                                                              #47
                                                                                              Originally posted by Theresa View Post
                                                                                              Ah come on Keane.
                                                                                              I was thinking about the sugar/sugar substitutes they use for flavouring and the potential for causing cravings, as well as the fact that chewing gum makes your stomach think it's about to get food so could cause hunger pangs.

                                                                                              I've gotten the answer that it's not an issue, so it's not an issue.

                                                                                              If I knew the answer I wouldn't have asked.

                                                                                              Comment


                                                                                                #48
                                                                                                Originally posted by Emmet View Post
                                                                                                Someone answer my metcon question please. Ended up throwing some EZ Curls and Crucifixes in last night, and my arms are in bits today, probably not the right exercises to be doing as part of that...
                                                                                                Will do
                                                                                                Originally posted by Emmet View Post
                                                                                                Followed by a "whatever's around" metcon.

                                                                                                So I've been setting up a barbell complex/metcon after every session. Just trying to figure out if there are any other things I can add in to mix it up, and make sure I'm hitting enough muscles.

                                                                                                Basically throw 20/25kg on the bar and do as many of these in a cycle until I am on the verge of puking.
                                                                                                Exercises include
                                                                                                Clean and Press (10)
                                                                                                Barbell Rows (15)
                                                                                                Bench Press (15)
                                                                                                Squats (15)
                                                                                                Jumps (to bench) (20)
                                                                                                Press Ups (15)
                                                                                                Burpees (10)
                                                                                                Thrusters (10) - 10kg is the biggest Medicine Ball / Kettlebell hybrid
                                                                                                Metcons come in all sorts of different flavours and styles. There's straught through chippers, 3/4/5 rounds for time, AMRAP in 20minutes, a single movement at high reps and intensity for time etc

                                                                                                There's plenty of sources of them, but Crossfit is going to be one of the4 msot common. They have daily workouts posted on their site (not all are metcons, but for the sake of handiness, I call them that). Their is suppsoed to be a structure and reasoning for all evcercises, but tbh there isn't. Not anymore. Some days the excercise put up is garbage. Design with no purpose other than being awkward.
                                                                                                I tend to ignore most of these unless it catches my eye.

                                                                                                Crossfit also have the named workouts (Fran, Jackie, Grace, Elizabeth, Helen etc). This are imo, much more structured and every so often one appears as the daily workout. Some of this are very tough, others aren't so tough, but you'll need to grind through.

                                                                                                Emmet, your metcon looks a bit confused. Halfway between two styles in a way.
                                                                                                If you want to do a metcon for say 3 rounds for time, then drop some of the excercises, and increase the load on the others, so that a round is tough to finish, it's still strength based training, not aerobics.

                                                                                                For example, Helen or Fran
                                                                                                SPOILER

                                                                                                Helen
                                                                                                ■400 meter run
                                                                                                ■16-24kg Kettlebell swing x 21
                                                                                                ■Pull-ups 12 reps
                                                                                                3 rounds for time

                                                                                                Fran
                                                                                                ■Thruster 95 lbs (change the weight to where you can do 10-15 reps in a single go)
                                                                                                ■Pull-ups
                                                                                                21-15-9 reps, for time


                                                                                                Or if you want to do high rep stuff. Look at the likes of Barbara or Cindy.

                                                                                                SPOILER

                                                                                                Barbara
                                                                                                ■20 Pull-ups
                                                                                                ■30 Push-ups
                                                                                                ■40 Sit-ups
                                                                                                ■50 Squats
                                                                                                3 min rest
                                                                                                3-5 rounds for time

                                                                                                Cindy
                                                                                                ■5 Pull-ups
                                                                                                ■10 Push-ups
                                                                                                ■15 Squats
                                                                                                As many rounds as possible in 20 min

                                                                                                (looks easy, its not)


                                                                                                For the time being, I'd stick with Crossfit's named workouts (see here. Some will be off limts for a while). Pick one, do it, and then do it again a month or two later. Try to improve your time/score.
                                                                                                There are plenty of non-crossfit metcons out there that are fine. But unless you are looking for something sports specific, there's enough in that list to get through.

                                                                                                Comment


                                                                                                  #49
                                                                                                  Cool cheers,

                                                                                                  thing is though, these have actually been going really well. I do my standard 3x5 instead of 5x5 beforehand, and then burnout in the gym doing the exercises above.

                                                                                                  There's no place to run in the gym bar the treadmills, which I have no interest in, and I reckon I'm getting fitter doing the barbell exercises than I would without. They're definitely improving my lifting too, as hitting PBs even on a cut.

                                                                                                  I tried to develop a kind of mini all body routine to just get the last few gasps out of you, while still working the big muscle groups. I'm still flailing miles behind on some exercises too, can't get more than 1 pull up really, and got no cash to get myself a pull up bar for continual practise at home.

                                                                                                  With work now too, I can't be spending ages in the gym, so 3x5 and then into the above (takes no more than 10mins until whiteface / spins / dizzyness) usually takes less than 50/60 minutes. The "Tabatha Intervals" mentioned just below the WODs on that link seem close-ish to what I've been trying to do with my last energy reserves / gym time.

                                                                                                  I was just wondering if there's anywhere I'm not hitting with the above really. Though the gym facilities are great, you feel a bit of a plonker taking a bench, barbells, kettlebell and dumbbells for 10/12 minutes and getting setup. Was hoping to drop the dumbbells altogether and just keep a hold of the bar.

                                                                                                  Comment


                                                                                                    #50
                                                                                                    Just a quick question on diet, Im on that low card, no sugar lark and my meals are getting fairly monotonus at this stage. Anyways i havent had any soups and was just wondering if theres any available to buy for this kinda diet, or if anyone has any recipes for decent homemade ones?

                                                                                                    Comment


                                                                                                      #51
                                                                                                      Originally posted by Insanodude View Post
                                                                                                      Just a quick question on diet, Im on that low card, no sugar lark and my meals are getting fairly monotonus at this stage. Anyways i havent had any soups and was just wondering if theres any available to buy for this kinda diet, or if anyone has any recipes for decent homemade ones?
                                                                                                      Why soups?

                                                                                                      Go absolutely nuts on the spices for your meats instead. It will only get boring if you get lazy (from experience).

                                                                                                      Comment


                                                                                                        #52
                                                                                                        Originally posted by Emmet View Post
                                                                                                        Cool cheers,

                                                                                                        thing is though, these have actually been going really well. I do my standard 3x5 instead of 5x5 beforehand, and then burnout in the gym doing the exercises above.

                                                                                                        There's no place to run in the gym bar the treadmills, which I have no interest in, and I reckon I'm getting fitter doing the barbell exercises than I would without. They're definitely improving my lifting too, as hitting PBs even on a cut.

                                                                                                        I tried to develop a kind of mini all body routine to just get the last few gasps out of you, while still working the big muscle groups. I'm still flailing miles behind on some exercises too, can't get more than 1 pull up really, and got no cash to get myself a pull up bar for continual practise at home.

                                                                                                        With work now too, I can't be spending ages in the gym, so 3x5 and then into the above (takes no more than 10mins until whiteface / spins / dizzyness) usually takes less than 50/60 minutes. The "Tabatha Intervals" mentioned just below the WODs on that link seem close-ish to what I've been trying to do with my last energy reserves / gym time.

                                                                                                        I was just wondering if there's anywhere I'm not hitting with the above really. Though the gym facilities are great, you feel a bit of a plonker taking a bench, barbells, kettlebell and dumbbells for 10/12 minutes and getting setup. Was hoping to drop the dumbbells altogether and just keep a hold of the bar.
                                                                                                        There's nothign wrong with the excercises you are doing, that are all basic compound lifts. And coupling it with 3x5 is fine too. But I took it that your focus is getting stronger, the rep range you are doing isn't conducive to strength training. Just because you are getting every lasty gasp out of a muscle doesn't guarantee it'll benefit. It may even slow strength gains, its more suit to burning energy and/or fitness. Its bascially aerobics. What I was suggesting above was that you increase the load, reduce the weight and use power based anaerobic movements to get the same effect (whiteface / spins / dizzyness)

                                                                                                        The metcons I linked to were just examples of how keeping the list of movements shorter and simplier is jsut as good. It you can't do pull-ups then don't do those ones. There are loads yo can do to get the same effect.

                                                                                                        Don't worry about hitting every muscle, the ones you've listed hit everywhere multiple times.
                                                                                                        SPOILER
                                                                                                        Clean and Press - Legs, back, shoulders, triceps
                                                                                                        Barbell Rows - back, biceps or shoulders
                                                                                                        Bench Press - chest, triceps
                                                                                                        Squats - legs
                                                                                                        Jumps (to bench) - legs
                                                                                                        Press Ups - chest, triceps
                                                                                                        Burpees - legs, back
                                                                                                        Thrusters - legs, shoulders, tricep


                                                                                                        By the time you get to Thrusters, your legs are going to be shot. I rather reduce the excercises and hit everywhere once, but harder. Also the more excercises you have, the more you dick about getting set up.

                                                                                                        Here's a complex I put up in my log last year. Lower reps, higher weight, and high tempo. All you need is a barbell. Hand don't ever leave the bar (except to take plates off if you go for that option). It starts with the tougher movement, press, and works to deadlifts, where the load is less taxing.

                                                                                                        Complex - Military press, squat, bent-over rows, stiff leg dead lifts, 8 reps each, one after the other no rest, hands don't move from barbell.
                                                                                                        I started ith 40kg, then droped it to 30kg, then 20kg. The only rest I took was the 5 seconds or so to take two 5kg plates off, so its close to 100 reps with pretty much no rest.
                                                                                                        Really gets the heart rate up, next time i might try to keep it at 40kg for 3 rounds

                                                                                                        Comment


                                                                                                          #53
                                                                                                          I should say that I'm on a cut at the moment, and I'm all about maintaining strength, not quite strength building.

                                                                                                          So aerobics / cardio / fitness / burning energy is exactly the main goal of this for me, and the benefits of lifting things instead of running/cycling/rowing is a bonus...

                                                                                                          Comment


                                                                                                            #54
                                                                                                            Originally posted by Emmet View Post
                                                                                                            I should say that I'm on a cut at the moment, and I'm all about maintaining strength, not quite strength building.

                                                                                                            So aerobics / cardio / fitness / burning energy is exactly the main goal of this for me, and the benefits of lifting things instead of running/cycling/rowing is a bonus...
                                                                                                            You're up early!

                                                                                                            Im finding two days SS and one day high rep high intensity metcon style training is giving really good results. Look a lot more defined.
                                                                                                            This may or may not be an original thought of my own.
                                                                                                            All efforts were made to make this thought original but with the abundance of thoughts in the world the originality of this thought cannot be guaranteed.
                                                                                                            The author is not liable for any issue arising from the platitudinous nature of this post.

                                                                                                            Comment


                                                                                                              #55
                                                                                                              been up this early for last 3 weeks! Early bird catches the job apparently.

                                                                                                              Problem with the SS x2, Metcon x1 is that I'm gyming with others, none of whom are on a cut, so I lose a spotter if I fall out of sync with their routine. By throwing in this quasi-metcon-cardio yoke after the lifts, am I not getting the best of both worlds?

                                                                                                              Re: Definition, its all in the kitchen!

                                                                                                              Comment


                                                                                                                #56
                                                                                                                Originally posted by Emmet View Post
                                                                                                                Problem with the SS x2, Metcon x1 is that I'm gyming with others, none of whom are on a cut, so I lose a spotter if I fall out of sync with their routine. By throwing in this quasi-metcon-cardio yoke after the lifts, am I not getting the best of both worlds?
                                                                                                                All metcons are about fitness, cardio and conditioning. And I suppose active recovery.
                                                                                                                There isn't a huge differnece is what you are currently doing to what I suggested, I just think you are over complicating it with lots of exercises when half the amount would do.

                                                                                                                When on a cut, I'm the same as Thereas said, two days SS for strength, the 3rd high intensity metcons for burning energy and upping conditioning. Even every second session SS and Metcons. I generally find that my performance in both is better when I split them like this.
                                                                                                                If you are doing both, instead of getting the best of both worlds, you won't be able to give 100% on the cardio/fitness stuff after weights. and you won't consistantly lift well if you are fatiguing muscles the night before.
                                                                                                                If you feel you have enough rest before the next lifting session then go ahead. (but i'd still include some more intense stuff, and rower or treadmill sprints - totally different to mindless running)



                                                                                                                edit: changed it around a bit to cover your post above
                                                                                                                Last edited by Mellor; 07-07-11, 07:08.

                                                                                                                Comment


                                                                                                                  #57
                                                                                                                  Re:General fitness / weight loss questions.

                                                                                                                  READ THIS it's concise, articulate and absolutely nails it.

                                                                                                                  This Is Silly: An Outlook on Training, Part 1

                                                                                                                  By Matt BrownPublished: July 8, 2011

                                                                                                                  A colleague of mine, Sean Croxton of Underground Wellness, wrote a blog on his website titled “This is Silly.” That blog inspired this article, which basically encompasses everything silly in the confines of nutrition and the many different opinions out there.

                                                                                                                  An outlook on training concerning family

                                                                                                                  My dad recently came to visit me and my sister from our hometown. One way or another, as is usual when relatives or friends are around me, the topic of nutrition and training comes up. He said to me, “Your old man has been working out lately a couple days a week. I’ve been doing a step class that has resistance training intertwined in it and the other days are bodybuilding type splits.”

                                                                                                                  He didn’t use the terminology “bodybuilding splits,” but I threw that in there to summarize what he was in essence doing. At this point, I had a choice. I could say to him, “Dad, what you’re doing is a waste of time and this style of training won’t get you anywhere.” Or I could just sit back and smile and be glad my dad was striving to be active and humbling himself enough to talk about it with me. The fact that he was bringing it up to me tells me I have done something in him to keep him lifting and moving.

                                                                                                                  Look at your surroundings and your family. How many of them even do anything remotely resembling strength or conditioning work? Was I going to introduce my dad to block periodization, 5/3/1, or anything like that? Hell no. I’m happy he’s doing anything besides sitting on his ass drinking a bottle of wine every night to pass the time, which may be part of the skyrocketing estrogenic levels among men nowadays. I choose the latter and decided to just smile.

                                                                                                                  I took a sip of my water, looked at him, and asked him one question. “How is your body feeling?”

                                                                                                                  He said, “I feel great.”

                                                                                                                  I replied, “Perfect. Dad, I’m so glad you’re working out and being conscious about your health.”

                                                                                                                  That was our conversation. I listened and kept my mouth shut because it wasn’t about me. It was about him. The only time I chimed in when we talked about working out was if he mentioned that he was hurting. But he isn’t hurting. He’s happy with what he’s doing. Who am I to change a positive habit with my own ideas? It isn’t about you.

                                                                                                                  An outlook on training concerning clients


                                                                                                                  This kind of conversation may start when your client walks through the door. “I ran five miles yesterday trying to burn off this extra fat left on me and I only ate 500 calories today.” Huh?

                                                                                                                  First off, let me start by saying that our clients don’t view training as we do. It isn’t their entire lives. They do not eat, breathe, and live it like we do. Although we try to instill this in them for their personal success, what we must do is educate them based on their goals and find common ground. Some don’t even know what they’re asking for nor do they realize the commitment it entails. Are they willing to do what they’re asking for?

                                                                                                                  The one hourers: Some want to look shredded and a buck 85 like in high school, but they weren’t really that shredded then. What they remember from then compared to how they look now are two different things. Is what they’re asking for realistically based on their personality, work ethic, and discipline? Some just don’t give a shit. I should say in a more positive manner, they give you the hour and not anything more outside of it. Training is a stop on the train, an hour away from the kids, an hour outside of team practice, games, or school, an hour away from the job they despise. They are going to have their two glasses of wine at night, they’re going to network at clubs, and they’re going to be on the computer until two in the morning. They’re going to do what they want because it’s a behavior that they don’t want to leave, their crutch if you will.

                                                                                                                  I did not mention this because it’s a bad thing, but when they expect results without having to make sacrifices, this situation isn’t good.

                                                                                                                  The converted: They do well most of the time and occasionally say F it. I believe the majority of folks we see fall into this category if we’re doing a good job. The converted believe in you, listen to what you say, and are willing to make sacrifices to achieve their idea of success. They are conscious and choose to keep their eyes open to benefit their body.

                                                                                                                  The ass kickers: These folks are wired to not give in to normality, neither are they born to it. They eat clean, get to bed on time, and are in better shape than you if you’re doing your job correctly. These are the folks you train with to push yourself and challenge yourself. My 47-year-old female client can bust out 15 legit chin-ups. There aren’t too many guys doing that. Sack up guys. I must add a side note—she is a genetic freak anyways and crazy strong, so she would fall into the category of you work with them and try not to get them hurt. It’s like Buddy Morris said, “You don’t train Curtis Martin. You work with Curtis Martin.”

                                                                                                                  So what does this mean? This means choose your battles and don’t break your own heart. I learned this from my colleague Ryan Burgess. It pissed me off because I want all my clients to be ass kickers and not settle for anything but their best. However, if they don’t want to be ass kickers and are happy with breaking a sweat, so be it. Swallow your pride and ego. Let them be happy being them. You be happy being you. You have control over yourself and staying in the ass kicking zone, and when they decide to get serious, they can.

                                                                                                                  Things to think about for success

                                                                                                                  There are things in life that need to take place in order for them to work. Squat to poop, propose to marry, eat to live, read to learn, gravity to fall, strength to stand, and a hand to shake. There are some things in training that must be done in order for you to succeed and for you to keep your mind off the so-called rules.
                                                                                                                  • Practice what you preach. This is number one. Be the example.
                                                                                                                  • Keep it simple. What can we do that will provide the greatest effect to the body in the least amount of time.
                                                                                                                  • Lift or press something heavy. I learned this from Jason Ferrugia. Perform 1–5 sets of 1–5 reps on the main lifts followed by 6–15 reps on the assistance lifts, pushing to a density style training. Finish in an hour to an hour fifteen minutes.
                                                                                                                  • Training should be instinctive, not always planned. Listen to your body. If you randomly hit a set of deadlifts because your co-worker had 225 lbs on the bar and it feels like a piece of cake, work up to a heavy triple and call it. Don’t just say, “Well, I squatted two days ago and I shouldn’t deadlift” or better yet, “Even though my shoulders feel horrible, I’m going to try to bench.”
                                                                                                                  • The conjugate method is handy when…you understand it and have athletes under your guidance for multiple years, not two months.
                                                                                                                  • Do not buy into what others say is the next best thing. Folks, TRX is not a godsend and neither is a Bosu ball or kettlebells, although kettlebells are pretty useful. Look up Dan John.
                                                                                                                  • Warming up is great and so is stretching.
                                                                                                                  • Stop BSing yourself. If you don’t like the way you look, change it.
                                                                                                                  • Stay with a program. If you just purchased an ebook or whatever, stay with it for awhile—for at least 4–6 months. Don’ hop around doing randomly implemented programs. The best way to find out what works best in attaining your goals is to stay with it. A hybrid of programs would work such as 5/3/1 with three assistance exercises followed by a fat loss/Tabata/Strongman circuit. Just commit to it. Stop trying everything or better yet, stop doing nothing.
                                                                                                                  • Find a mentor. Currently, I’m learning from a man who is an absolute genius. He’s a smart man and a freaking beast! He trained, learned from, and knows the likes of Lee Haney, Matt Mendenhall, Fred Hatfield, Rachel McLish, Bertil Fox, Mike Christian, Bob Paris, and Richard Sorin. You need someone like this in your life. He or she can change everything you know about training and may just have an impact on who you become or know in this field.
                                                                                                                  • Perform squats. High rep squats work well for putting on mass. Don’t believe me? Look up Tom Platz repping out 500 lbs 23 times ass to grass. Use caution with these, as it will kick your ass for a week.
                                                                                                                  • Do hill sprints. These are the best exercise second to clean eating for fat loss.
                                                                                                                  • The author enjoys these and you should, too.


                                                                                                                  Lighten up. Dude, if you want a beer, drink one. That won’t make or break your training. If you club it three nights out of the week, this will destroy training. Know the difference.

                                                                                                                  Shut up and listen. You’ll be surprised what you will learn if you listen to others without trying to chime in. You may not agree with 95 percent of what comes out of their mouth, but the other 5 percent may actually help you progress your training. Nobody is right and nobody is wrong. The only people right are those who make the effort to try the things that were once neglected. Switch nonsense to commonsense and find common ground.

                                                                                                                  Shut up on the Paleo diet. Ha ha! I do believe it has validity but so does the notion of eating real food. If God made wheat, it won’t kill everyone and cause leaky gut. It will only cause damage for those who stress enough over gluten. Sweet potatoes are awesome post-workout. That’s all I will give you Abercrombie fiends. A good motto that Sean Croxton coined was J.E.R.F.—a shocker to be revealed later.

                                                                                                                  Go to the beach to recover and stretch. Some of you don’t have this option, which sucks. One of the best things you can do for recovery is to take a walk. Try this on the seashore to get your mind right and realize how little in life we need to enjoy it.

                                                                                                                  Compete or watch a rugby match. This is how our youth ought to be raised—with some balls and collisions. That sounded bad, but it’s kind of funny so I’ll leave it for the next thing to think about.

                                                                                                                  Don’t take life or training too seriously. This is something I’ve struggled with personally. I love training, and when it comes time to lift, it is time to stay out of my way. This is how I’m wired. But here is the deal—you need to have fun every now and then. Lighten up, man. Play some pickup games for a warm up, bust your training partners’ balls, and enjoy your ability to learn from the best on this website. Life is too short, man. It can be taken away in a second. Know when to turn on the attitude and know when to smile, breath, and enjoy what you have.

                                                                                                                  Girls need to lift heavy as well. If you want to be skinny fat, then run. If you want to be lean, then lift heavy and condition. If you want a softer look, I have no idea what to tell you other than watch how often you lift and drink more wine.

                                                                                                                  Comment


                                                                                                                    #58
                                                                                                                    I'm proud to say that I am very close to being in the 'ass-kickers' bracket. Good article.

                                                                                                                    Comment


                                                                                                                      #59
                                                                                                                      Suggested diet on boards.ie today.

                                                                                                                      Internet is choc full of droolers imo

                                                                                                                      Comment


                                                                                                                        #60
                                                                                                                        Originally posted by Emmet View Post
                                                                                                                        Suggested diet on boards.ie today.

                                                                                                                        Internet is choc full of droolers imo

                                                                                                                        Whats that the diabetes diet?

                                                                                                                        Comment

                                                                                                                        Working...
                                                                                                                        X